User avatar
 
ohthatpatrick
Thanks Received: 3806
Atticus Finch
Atticus Finch
 
Posts: 4661
Joined: April 01st, 2011
 
 
 

Re: Q9 - Museum visitor: The national government

by ohthatpatrick Sat Jan 20, 2018 1:50 am

Yeah, I agree.

I don't know who that "ohthatpatrick" was that wrote that. :)

Well ... I mostly agree. First of all, as we've all been saying, we hate this question.

Secondly, there was no reason for me to say that "revenue does not exceed expenses" is a premise supporting the idea that "the mandate will increase the museum's operating expenses".

Where I was being correct was in saying that you could support "the mandate will increase operating expenses" with the fact that "the mandate involves a 5% increase to minimum wage paid workers".

And while I would not have identified "the mandate will increase operating expenses" as an intermediate conclusion as I was reading the argument (they didn't give us any of the usual structural signs that they consider this an intermediate conclusion), I could ultimately make peace with the idea that this is an opinion that is fair game when it comes to objecting to the argument.

Basically, anything in the future tense is technically an opinion.

Check out this question, which has a similar attack point.
https://www.manhattanprep.com/lsat/foru ... 14559.html

And when you're doing Nec Assump, just ask yourself, "Which answer, if negated, most weakens"? I don't think I've ever seen a question where the winner of that analysis wasn't the correct answer.
 
MahmoudA830
Thanks Received: 0
Vinny Gambini
Vinny Gambini
 
Posts: 3
Joined: August 25th, 2019
 
 
 

Re: Q9 - Museum visitor: The national government

by MahmoudA830 Thu Aug 06, 2020 10:25 pm

What is really bothering me here is the idea that "significantly more" is not less than 5% more even though technically we don't know that. Let's say that significantly more here is 4% more, and we took the negation of (A), then the argument would still not fail. Are we really allowed to assume that "significantly more" here is actually what we need it to mean?

This question is really messing with my perception of what we're allowed to assume and what we're not because a lot of other incorrect answers trade on assumptions that are seemingly harmless when in fact they're not.
 
Laura Damone
Thanks Received: 94
Atticus Finch
Atticus Finch
 
Posts: 468
Joined: February 17th, 2011
 
 
 

Re: Q9 - Museum visitor: The national government

by Laura Damone Tue Aug 11, 2020 1:17 pm

As has been stated widely on this thread, this problem stinks. It stinks like a skunk eating boiled cabbage.

However, a decent measure of whether you are allowed to assume something on the LSAT is whether it is factually true.

Thinking about minimum wage, that's 15/hr if you're lucky. What's 5% of that? A whopping 75 cents. I don't think in any universe we can say that making 15.75 is significantly more than making 15 even. 4% of 15/hr is even worse: 60 cents.

Basically, the museum visitor's argument is the same one that the opponents make against the fight for a 15 movement: paying folks a higher minimum wage is going to mess up businesses. Well, this is only true for businesses who pay people the minimum wage or darn close to it. In other words, the argument assumes that, in these businesses, people are making a low enough wage that it would have to increase with the new mandate.

Hope this helps!
Laura Damone
LSAT Content & Curriculum Lead | Manhattan Prep